Question
1) Determine if w is in the subspace spanned by v1, v2, v3
2) Are the vectors v1, v2, v3 linearly dependent or independent? justify your answer
Question 2. (15 pts) Let vi=(-3 0 6), v2= (-2 2 3], V3= (0 - 6 37, and w= [1 11 9. (1). Determine if w is in the subspace
0 0
Add a comment Improve this question Transcribed image text
Answer #1

2 2 Vi = 13] U2= 2 V3 = -6 3 3 Let wi Civ, +C₂ V2 + (₂ V3. o • 6:40 - 3 In augmented form, we have. 3 o 2 14 6 3 3 9 Applyingwis not in the subspace spanned by V1, V2, V3. putting vi, U2, V3 matrix A. column in - 3 - 2 A = 2 6 3 3 Now determinant of

Add a comment
Know the answer?
Add Answer to:
1) Determine if w is in the subspace spanned by v1, v2, v3 2) Are the...
Your Answer:

Post as a guest

Your Name:

What's your source?

Earn Coins

Coins can be redeemed for fabulous gifts.

Not the answer you're looking for? Ask your own homework help question. Our experts will answer your question WITHIN MINUTES for Free.
Similar Homework Help Questions
  • Question 2. (15 pts) Let Vi= (-3 0 6)", v2= (-2 2 3)", V3= [0 -...

    Question 2. (15 pts) Let Vi= (-3 0 6)", v2= (-2 2 3)", V3= [0 - 6 3)", and w= [1 14 9)? (1). Determine if w is in the subspace spanned by V1, V2, V3. (2). Are the vectors Vi, V2, V3 linearly dependent or independent? Justify your answer.

  • Question 2. (15 pts) Let vi= (-3 0 6)", V2= (-2 2 317, V3= [0 -...

    Question 2. (15 pts) Let vi= (-3 0 6)", V2= (-2 2 317, V3= [0 - 6 3)", and w=(1 14 9) (1). Determine if w is in the subspace spanned by va, V2, V3. (2). Are the vectors V1, V2, V3 linearly dependent or independent? Justify your answer.

  • Let v1= [−3 0 6]T , v2= [−2 2 3]T , v3= [0 − 6 3]T...

    Let v1= [−3 0 6]T , v2= [−2 2 3]T , v3= [0 − 6 3]T , and w= [1 14 9]T . (1). Determine if w is in the subspace spanned by v1, v2, v3. (2). Are the vectors v1, v2, v3 linearly dependent or independent? Justify your answer.

  • Question 2. (15 pts) Let vi= [-3 0 6)". Vy=[-2 2 3". Vg= [0 - 6...

    Question 2. (15 pts) Let vi= [-3 0 6)". Vy=[-2 2 3". Vg= [0 - 6 3), and w=[1 14 97 (1). Determine if w is in the subspace spanned by V. V2 V3. (2). Are the vectors Vi, V2, V3 linearly dependent or independent? Justify your answer.

  • Can I get help with questions 2,3,4,6? be the (2) Determine if the following sequences of vectors vi, V2, V3 are linear...

    Can I get help with questions 2,3,4,6? be the (2) Determine if the following sequences of vectors vi, V2, V3 are linearly de- pendent or linearly independent (a) ces of V 0 0 V1= V2 = V3 = w. It (b) contains @0 (S) V1= Vo= Va (c) inations (CE) n m. -2 VI = V2= V3 (3) Consider the vectors 6) () Vo = V3 = in R2. Compute scalars ,2, E3 not all 0 such that I1V1+2V2 +r3V3...

  • -9 2. Let Vi-8.V2,andvs-2, let B -(V,V2,Vs), and let W be the subspace spanned , let B -(Vi,V2,V3...

    -9 2. Let Vi-8.V2,andvs-2, let B -(V,V2,Vs), and let W be the subspace spanned , let B -(Vi,V2,V3), and let W be the subspace spanned by B. Note that B is an orthogonal set. 17 a. 1 point] Find the coordinates of uwith respect to B, without inverting any matrices or L-2 solving any systems of linear equations. 35 16 25 b. 1 point Find the orthogonal projection of to W, without inverting any matrices or solving any systems of...

  • 1. Find A if (2A)' = [ : :] 2. Determine if {(x,y) : x -...

    1. Find A if (2A)' = [ : :] 2. Determine if {(x,y) : x - y = 1) is a subspace of vector space V - R 3. Let vi, V2, V3 be three linearly independent vectors in a vector space V. Is the set {v1 - 2v2, 2v2 - 3v3, 3V3 - Vi} linearly independent or linearly dependent? Prove your answer.

  • #8. Let W be the subspace of R3 spanned by the two linearly independent vectors v1...

    #8. Let W be the subspace of R3 spanned by the two linearly independent vectors v1 = (-1,2,2) and v2 = (3, -3,0). (a) Use the Gram-Schmidt orthogonalization process to find an orthonormal basis for W. (b) Use part (a) to find the matrix M of the orthogonal projection P: R W . (c) Given that im(P) = W, what is rank(M)?

  • Problem #18: [2 marks] Let W be the subspace of R4 spanned by the vectors u...

    Problem #18: [2 marks] Let W be the subspace of R4 spanned by the vectors u - (1,0,1,0), u2 = (0.-1, 1.0), and ug = (0.0, 1,-1). Use the Gram-Schmidt process to transform the basis (uj, u, uz) into an orthonormal basi (A) v1 = (-12,0, 2.0), v2 - (VG VG VG, o), v3 - (I ) (B) v1 = (-V2.0, .), v2 - (VG VG VG o), v3 - (™J - V3 VI-V3) (C) v1 - ($2.0, 92.0), v2...

  • Consider the following three vectors in ; v1 = (1, 7, −2), v2 = (4, 3,...

    Consider the following three vectors in ; v1 = (1, 7, −2), v2 = (4, 3, 5), v3 = (2, −11, 9): i) Say whether v1, v2, v3 are linearly dependent or linearly independent. (Justify) ii) Say if v1, v2, v3 generate . (justify) iii) If it exists, determine the constants c1, c2, c3, such that c1v1 + c2v2 + c3v3 = (0, −5, 13/5), or argue why it cannot be written as a linear combination. We were unable to...

ADVERTISEMENT
Free Homework Help App
Download From Google Play
Scan Your Homework
to Get Instant Free Answers
Need Online Homework Help?
Ask a Question
Get Answers For Free
Most questions answered within 3 hours.
ADVERTISEMENT
ADVERTISEMENT
ADVERTISEMENT